Sei sulla pagina 1di 11

Sistemas No Lineales.

Tarea #1
Alumno: Jos Juan Rincn Pasaye
26 de enero de 2006
1 Problema #1
Sea el sistema no lineal siguiente
x = f(x; t); x(t
0
) = x
0
; x(t) R
n
demuestre que este sistema siempre se puede reescribir como un sistema autnomo. Sugerencia: aumente el vector
de estado x(t) con x
n+1
(t)
M
= t, es decir, el nuevo vector de estado z(t)
M
=
_
x(t)
t
_
=
_
x(t)
x
n+1
(t)
_
R
n+1
, etc.
1.1 Solucin
Con la denicin del nuevo estado, obtenemos

z(t)
M
=
_

x(t)
1
_
=
_
f(x; t)
1
_
=
_
f(x; x
n+1
)
1
_
=
_
f(z)
1
_
Es decir,

z(t) =
_
f(z)
1
_
M
= F(z)
Con lo cual desaparece la dependencia explcita del tiempo en F(z) por lo tanto se puede considerar como un sistema
autnomo.
2 Problema #2
Ejercicio 1.7 del Khalil, pag. 25 La gura muestra una interconexin en retroalimentacin de un sistema lineal invariante
en el tiempo con un elemento no-lineal variante en el tiempo. Las variables r, y y u son vectores de las mismas
dimensiones, (t; y) es una funcin valuada vectorialmente. Encuentre un modelo de espacio de estado teniendo a r
como entrada y a y como salida.
G(s)=C(sI-A)
-1
B
(t,y)
y
z
r u +
-
2.1 Solucin
El correspondiente modelo de estado para G(s) es el siguiente

x(t) = Ax(t) +Bu(t)


y = Cx(t)
de la gura se obtiene lo siguiente
u = r z = r (t; y)
sustituyendo en la ecuacin de estado, obtenemos
1

x(t) = Ax(t) +B(r (t; y)) = Ax(t) B(t; y)) +Br;


pero y = Cx, por lo tanto, un modelo de espacio (no lineal) de estado es el siguiente

x = Ax B(t; Cx) +Br


y = Cx
3 Problema #3
Ejercicio 1.16 del Khalil, pags. 29 y 30: La gura muestra un diagrama esquemtico de un sistema oscilador
transalcional con actuador rotatorio (TORA). El sistema consiste en una plataforma de masa M conectada a un marco
de referencia jo por un resorte lineal con constante k. La plataforma puede moverse slo en el plano horizontal, paralelo
al eje del resorte. Sobre la plataforma, una masa rotatoria m de prueba es actuada por un motor de CD, esta masa
tiene un momento de inercia I alrededor de su centro de masa, localizado a una distancia L de su eje de rotacin. El
torque de control aplicado a la masa de prueba es denotado por u:La masa de prueba rotacional crea una fuerza que
puede ser controlada para amortiguar el movimiento translacional de la plataforma. Derivaremos un modelo para el
sistema, despreciando la friccin. La gura muestra que la masa de prueba est sujeta a fuerzas F
x
y F
y
y a un torque
u. Escribiendo las leyes de Newton en el centro de masa y tomando momentos respecto al centro de masa se obtienen
las siguientes ecuaciones
m
d
2
dt
2
(x
c
+Lsin) = F
x
(1)
m
d
2
dt
2
(Lcos ) = F
y
(2)
I

= u +F
y
Lsin F
x
Lcos (3)
Donde es la posicin angular de la masa de prueba (medida en sentido antihorario). La plataforma est sujeta a
las fuerzas F
x
y F
y
en direcciones opuestas, as como a la fuerza de restauracin del resorte. Las leyes de Newton para
la plataforma nos dan
M

x
c
= F
x
kx
c
(4)
Donde x
c
es la posicin translacional de la plataforma.
M
k
x
c
u F
x
L
m
F
y

a) Derivando y eliminando Fx y F
y
muestre que las ecuaciones de movimiento se reducen a
2
D()
_

x
c
_
=
_
u
mL

sin kx
c
_
; donde D() =
_
I +mL
2
mLcos
mLcos M +m
_
b) Resolviendo la ecuacin anterior para

y para

x
c
, muestre que
_

x
c
_
=
1

_
M +m mLcos
mLcos I +mL
2
_
_
u
mL

sin kx
c
_
donde
= (I +mL
2
)(m+M) m
2
L
2
cos
2
_ (I +mL
2
)M +mI > 0
c) Usando x
1
= ; x
2
=

; x
3
= x
c
; y x
4
=

x
c
como las variables de estado y u como la entrada de control, escriba
las ecuaciones de estado.
d) Encuentre todos los puntos de equilibrio del sistema.
3.1 Solucin
a) Calculando la derivada indicada en (1)
m

x
c
mLsin

+mLcos

= F
x
(5)
En forma similar, al calcular la derivada indicada en (2)
mLcos

mLsin

= F
y
(6)
Sustituyendo (5) y (6) en (3)
I

= u + (mLcos

mLsin

)Lsin (m

x
c
mLsin

+mLcos

) cos
I

= u mL
2
sin cos

mL
2
sin
2

x
c
Lcos +mL
2
sin cos

mL
2
cos
2

simplicando
I

= u mL
2

(sin
2
+ cos
2
) m

x
c
Lcos
I

= u mL
2

x
c
Lcos
despejando
(I +mL
2
)

+mLcos

x
c
= u (7)
Ahora sustituimos (5) en (4) y obtenemos
M

x
c
= (m

x
c
mLsin

+mLcos

) kx
c
reacomodando la ecuacin
mLcos

+ (M +m)

x
c
= mLsin

kx
c
(8)
3
Las ecuaciones (7) y (8) son el modelo pedido
b) Para mostrar el resultado pedido basta con obtener la inversa de D() :
D() =
_
I +mL
2
mLcos
mLcos M +m
_
por lo tanto
D
1
() =
_
I +mL
2
mLcos
mLcos M +m
_
1
=
1

_
M +m mLcos
mLcos I +mL
2
_
donde
= (I +mL
2
)(M +m) m
2
L
2
cos
2
:
que es el resultado esperado. Por otro lado, desarrollando el producto podemos obtener
= (I +mL
2
)M +mI +m
2
L
2
m
2
L
2
cos
2
:
= (I +mL
2
)M +mI +m
2
L
2
(1 cos
2
)
= (I +mL
2
)M +mI +m
2
L
2
sin
2

pero como 0 _ sin


2
_ 1
_ (I +mL
2
)M +mI
y como m, L, y M son positivas
> 0
c) Con las variables de estado sugeridas x
1
= ; x
2
=

; x
3
= x
c
; y x
4
=

x
c
se obtiene

x
1
= x
2

x
2
=
1

_
kmLx
3
cos x
1
m
2
L
2
x
2
sinx
1
cos x
1
+ (m+M)u

x
3
= x
4

x
4
=
1

_
(I +mL
2
)(mLx
2
sinx
1
kx
3
) mLcos x
l
u

d) Para hallar los puntos de equilibrio del sistema igualamos las derivadas de los estados a cero, obteniendo
0 = x
2
0 =
1

_
kmLx
3
cos x
1
m
2
L
2
x
2
sinx
1
cos x
1
+ (m+M)u

0 = x
4
0 =
1

_
(I +mL
2
)(mLx
2
sinx
1
kx
3
) mLcos x
l
u

o bien,
x
2
= 0; x
4
= 0;
kmLx
3
cos x
1
+ (m+M)u = 0 (9)
(I +mL
2
)kx
3
+mLcos x
l
u = 0 (10)
despejando x
3
de (10)
4
x
3
=
mLcos x
1
u
k(I +mL
2
)
sustituyendo en (9)

m
2
L
2
cos x
1
u
(I +mL
2
)
cos x
1
+ (m+M)u = 0
despejando
cos
2
x
1
=
(m+M)
m
2
L
2
(I +mL
2
)
de donde
x
1
= cos
1
_

_
(m+M)
m
2
L
2
(I +mL
2
)
_
= cos
1
_

_
(m+M)
m
2
L
2
(I +mL
2
)
_
+ 2n; con n = 0; 1; 2; :::
es decir, se tiene una innidad de puntos de equilibrio (dos para cada valor de n), sin embargo, como x
3
es un ngulo,
fsicamente estos equivalen solamente a cuatro:
x
1
=

; x
1
= (

)
donde

M
=
_
cos
1
_
(m+M)
m
2
L
2
(I +mL
2
); restringido a 0 _

_
_
sustituyendo estos valores de x
1
en x
3
obtenemos
x
3
=
mL
_
(m+M)
m
2
L
2
(I +mL
2
) u
k(I +mL
2
)
=
u
k
_
m+M
I +mL
2
es decir (en resumen), los cuatro puntos de equilibrio son:
x
2
= x
4
= 0;
x
1
=

; x
3
=
u
k
_
m+M
I +mL
2
x
1
= (

); x
3
=
u
k
_
m+M
I +mL
2
4 Problema #4
Ejercicio 2.4 del Khalil, pag. 78. Sugerencia: Use pptoool.m o pplane5.m de Matlab. Para cada uno de los
siguientes sistemas, construya el plano de fase y discuta el comportamiento cualitativo del sistema.
1)

x
1
= x
2

x
2
= x
1
2 tan
1
(x
1
+x
2
)
2)

x
1
= x
2

x
2
= x
1
+x
2
(1 3x
2
1
2x
2
2
)
3)

x
1
= 2x
1
x
1
x
2

x
2
= 2x
2
1
x
2
4)

x
1
= x
1
+x
2
x
1
([x
1
[ +[x
2
[)

x
2
= 2x
1
+x
2
x
2
([x
1
[ +[x
2
[)
5
4.1 Solucin
1) El correspondiente plano de fase obtenido mediante pplane se muestra en la gura siguiente
x ' = y
y ' = x - 2 atan(x + y)
-4 -3 -2 -1 0 1 2 3 4
-4
-3
-2
-1
0
1
2
3
4
x
y
Comentarios: El sistema posee 3 puntos de equilibrio (resaltados en la gura): dos puntos silla en (2.33,0) y un
nodo estable en el origen. Se puede apreciar tambin que cualquier condicin inicial producir una trayectoria inestable,
excepto las que estn delimitadas por las rectas paralelas de pendiente (-1) que cruzan por los dos puntos silla del plano
de fase. las cuales convergen hacia el origen.
En este sistema no puede haber oscilaciones, las trayectorias o convergen al origen o crecen de manera no acotada
2) En la gura siguiente se muestra el plano de fase correspondiente
x ' = y
y ' = - x + y (1 - 3 x
2
- 2 y
2
)
-2 -1.5 -1 -0.5 0 0.5 1 1.5 2
-2
-1.5
-1
-0.5
0
0.5
1
1.5
2
x
y
Comentarios:El sistema posee solamente un punto de equilibrio (el origen). el cual es una espiral inestable, sin
embargo, observando el plano de fase se v que solamente las trayectorias que comienzan en el interior de la regin con
forma de "valo" se alejan del origen, pero permanecen acotadas todo el tiempo dando vueltas sobre el permetro de
dicho valo. En cambio, las trayectorias que comienzan fuera del valo son atradas hacia l, permaneciendo tambin
6
atrapadas dando vueltas sobre el permetro del valo. aparentemente dicho valo es una "rbita atractora". En este
sistema todas las trayectorias son oscilatorias (excepto la trivial x
1
= x
2
= 0) y se mantienen acotadas.
3) A continuacin se muestra el plano de fase correspondiente
x ' = 2 x - x y
y ' = 2 x
2
- y
-5 -4 -3 -2 -1 0 1 2 3 4 5
-6
-4
-2
0
2
4
6
x
y
Comentarios:El sistema tiene tres puntos de equilibrio: un punto silla en el origen y dos espirales estables en (1,2).
Cualquier trayectoria que comienza en el semiplano izquierdo tiende al punto (-1,2) conforme el tiempo crece y cualquier
trayectoria que comienza en el semiplano derecho converge al punto (1,2)
Todas las trayectorias son oscilatorias decrecientes, excepto las que inician sobre el eje vertical, estas convergen al
origen, pero sin presentar oscilaciones (esto se pudo observar estableciendo condiciones iniciales precisas sobre el eje
vertical usando valores introducidos por teclado).
4) Se muestra a continuacin el plano de fase
x ' = x + y + x (abs(x) + abs(y))
y ' = - 2 x + y - y (abs(x) + abs(y))
-2 -1.5 -1 -0.5 0 0.5 1 1.5 2
-2
-1.5
-1
-0.5
0
0.5
1
1.5
2
x
y
7
Comentarios: El sistema tiene tres puntos de equilibrio: dos puntos silla en (-0.464,1.2679) y en (0.464,-1.2679) y
una espiral inestable en el origen. Todas las trayectorias del sistema son inestables, en las regiones cercanas al origen ni
siquiera es fcil predecir si al crecer con el tiempo las trayectorias lo harn hacia un valor positivo o hacia uno negativo
Ninguna de las trayectorias presenta oscilaciones sostenidas por la rapidez con que las stas se alejan del origen.
5 Problema #5
Considere el sistema siguiente

x
1
= (x
1
x
2
1
) + 1 x
1
x
2

x
2
= (x
2
x
2
2
) + 1 x
1
x
2
5.1 Solucin
Primeramente calculamos los puntos de equilibrio igualando las derivadas a cero:
(x
1
x
2
1
) + 1 x
1
x
2
= 0
(x
2
x
2
2
) + 1 x
1
x
2
= 0
reescribiendo
(1 x
1
)x
1
+ 1 x
1
x
2
= 0
(1 x
2
)x
2
+ 1 x
1
x
2
= 0
factorizando
(1 x
1
)
2
= x
2
(11)
(1 x
2
)
2
= x
1
(12)
Sustituyendo x
1
de la ecuacin (12) en la ecuacin (11) obtenemos
_
1 (1 x
2
)
2

2
= x
2
Simplicando para x
2
,= 0 obtenemos
x
3
2
4x
2
2
+ 4x
2
1 = 0
factorizando
(x
2
1)(x
2
2
3x
2
+ 1) = 0
de donde obtenemos las siguientes tres soluciones:
x
2
= 1; x
2
=
3
_
5
2
Sustituyendo x
2
= 1, en (12) obtenemos x
1
= 0
y sustituyendo x
2
=
3
_
5
2
en (12), obtenemos: x
1
= x
2
=
3
_
5
2
por otro lado, si x
2
= 0; de la ecuacin (12) obtenemos
x
1
= 1
de donde hemos obtenido cuatro puntos de equilibrio:
8
p
1
= (0; 1); p
2
= (1; 0); p
3
= (
3 +
_
5
2
;
3 +
_
5
2
); p
4
= (
3
_
5
2
;
3
_
5
2
)
o aproximadamente
p
1
= (0; 1); p
2
= (1; 0); p
3
= (2:618; 2:618); p
4
= (0:3819; 0:3819):
Para determinar el tipo de puntos de equilibrio obtenemos el Jacobiano del sistema
J(x) =
_
2 + 2x
1
1
1 2 + 2x
2
_
Para p
1
= (0; 1), obtenemos
J(p
1
) =
_
2 1
1 0
_
cuyos valores propios son :
_
2 1 > 0;
_
2 1 < 0 y cuyos vectores propios correspondientes son:
_

_
2 + 1
1
_
;
_ _
2 + 1
1
_
,
de donde ste corresponde a un punto silla con espacios propios dados por las rectas:
x
2
=
1

_
2 + 1
x
1
- 2:414x
1
y x
2
=
1
_
2 + 1
x
1
- 0:414x
1
, respectivamente.
Para p
2
= (1; 0), obtenemos
J(p
2
) =
_
0 1
1 2
_
,
cuyos valores propios son :
_
2 1 > 0;
_
2 1 < 0 y cuyos vectores propios correspondientes son:
_

_
2 1
1
_
;
_ _
2 1
1
_
,
de donde ste corresponde a un punto silla con espacios propios dados por las rectas:
x
2
=
1

_
2 1
x
1
- 0:414x
1
y x
2
=
1
_
2 1
x
1
- 2:414x
1
, respectivamente.
Para p
3
= (
3 +
_
5
2
;
3 +
_
5
2
), obtenemos
J(p
3
) =
_
1 +
_
5 1
1 1 +
_
5
_
cuyos valores propios son :
_
5 > 0; y
_
5 + 2 > 0 y cuyos vectores propios correspondientes son:
_
1
1
_
;
_
1
1
_
;
de donde este punto corresponde a un nodo inestable con espacios propios dados por las rectas:
x
2
= x
1
y x
2
= x
1
, respectivamente.
Para p
4
= (
3
_
5
2
;
3
_
5
2
), obtenemos
9
J(p
4
) =
_
1
_
5 1
1 1
_
5
_
,
cuyos valores propios son :
_
5 < 0; y 2
_
5 < 0 y cuyos vectores propios correspondientes son:
_
1
1
_
;
_
1
1
_
,
de donde este punto corresponde a un nodo estable con espacios propios dados por las rectas:
x
2
= x
1
y x
2
= x
1
, respectivamente.
Las trayectorias en el plano de fase se pueden dibujar aproximadamente marcando los cuatro puntos de equilibrio y
dibujando sobre ellos las rectas que denen los espacios propios para despus trazar sobre una regin cercana a ellos el
comportamiento tpico de las trayectorias, posteriormente se unen de la manera ms continua posible las trayectorias
de una regin a otra.
A continuacin se muestra el trazo aproximado (hecho a mano) y el realizado mediante pplane, en ambos se
muestran las rectas correspondientes a los espacios propios para cada punto de equilibrio.
-0.5 0 0.5 1 1.5 2 2.5 3
-0.5
0
0.5
1
1.5
2
2.5
3
x
y
10
x ' = - (x - x
2
) + 1 - x - y
y ' = - (y - y
2
) + 1 - x - y
-0.5 0 0.5 1 1.5 2 2.5 3
-0.5
0
0.5
1
1.5
2
2.5
3
x
y
Comentarios: Del plano de fase se observan varias cosas interesantes:
- El plano de fase es simtrico con respecto a la recta x
1
= x
2
.
- Existen tanto trayectorias estables como inestables
- Se puede delimitar aproximadamente una regin de trayectorias estables como aquellas que comienzan dentro del
sector angular delimitado por las rectas que unen p
1
con p
3
y p
2
con p
3
. (hacia abajo de p
3
). Todas estas trayectorias
convergen al nico nodo estable (p
4
).
- El resto de las trayectorias son inestables
- Este sistema no posee ninguna trayectoria oscilatoria.
11

Potrebbero piacerti anche